Last visit was: 24 Apr 2024, 06:13 It is currently 24 Apr 2024, 06:13

Close
GMAT Club Daily Prep
Thank you for using the timer - this advanced tool can estimate your performance and suggest more practice questions. We have subscribed you to Daily Prep Questions via email.

Customized
for You

we will pick new questions that match your level based on your Timer History

Track
Your Progress

every week, we’ll send you an estimated GMAT score based on your performance

Practice
Pays

we will pick new questions that match your level based on your Timer History
Not interested in getting valuable practice questions and articles delivered to your email? No problem, unsubscribe here.
Close
Request Expert Reply
Confirm Cancel
SORT BY:
Date
Tags:
Show Tags
Hide Tags
User avatar
Director
Director
Joined: 28 May 2005
Posts: 986
Own Kudos [?]: 3460 [43]
Given Kudos: 0
Location: Dhaka
Send PM
Most Helpful Reply
e-GMAT Representative
Joined: 02 Nov 2011
Posts: 4346
Own Kudos [?]: 30781 [23]
Given Kudos: 635
GMAT Date: 08-19-2020
Send PM
General Discussion
Retired Moderator
Joined: 27 Aug 2012
Posts: 1015
Own Kudos [?]: 4054 [1]
Given Kudos: 156
Send PM
e-GMAT Representative
Joined: 02 Nov 2011
Posts: 4346
Own Kudos [?]: 30781 [3]
Given Kudos: 635
GMAT Date: 08-19-2020
Send PM
Re: Despite improvements in treatment for asthma, the death rate from this [#permalink]
3
Kudos
Expert Reply
bagdbmba wrote:
Hi Chiranjeev,
I'm not able to get it clearly...!

As you've mentioned (rather as concluded) "since A and B are not the possible reason, C is the cause of increased deaths."..than how can we say that 'only three given reasons are possible' as A & B are already discarded by the author and that's why C comes up as the conclusion...Right?

Please help!


Hi,

I don't really understand your query. I will still try to explain why option E is correct, but if the below explanation does not help, then please explain your doubt a bit more clearly.

Suppose I give you the following argument:

There are two possible explanations for Joe's success. One explanation is that he is super-smart. Other explanation is that he is very hard working. However, Tim is equally successful, but Tim is neither super-smart nor very hard working. Therefore, the actual reason for Joe's success is his ability to understand test maker's mindset.

What is the assumption in this argument?

The argument first tells two possible explanations, A and B, for Joe's success. But then, it says that there is Tim for whom neither explanation works.

Then, the argument concludes that C is the actual explanation for Joe's success.

In other words, the argument presents two explanations A and B for Joe's success and then, it indicates that these explanation could not explain Joe's success since they do not apply to Tim who is as successful as Joe. On the basis of this, the argument concludes that C is the reason for Joe's success.

Now, the first question that should comes to mind on reading the argument is that why not D, E, F, G, H etc could be reasons for Joe's success. Why does the author not consider these other possible explanations?

The reason is that the author assumes that only A, B, and C are possible explanations. So, when he sees that A and B cannot explain, he concludes that C is the reason.

Now, if you understand till now, then you can see that the argument I have constructed is very much parallel to the given argument and the same reasoning applies to explain why option E is correct.

bagdbmba wrote:
P.S: I actually chose D - as you've said "this statement also is not absolutely required for the conclusion to hold true" , I guess this sort of reasoning is pretty uncommon in GMAT CR... Let me know whether I'm wrong.


I don't think this kind of reasoning is uncommon in GMAT CR. You do have strengtheners in assumptions questions in GMAT questions. These strengtheners strengthen the argument but are not absolutely required for the conclusion to hold true.

Thanks,
Chiranjeev
Current Student
Joined: 19 Mar 2016
Posts: 27
Own Kudos [?]: 25 [0]
Given Kudos: 43
Location: India
Concentration: Leadership, Strategy
GMAT 1: 630 Q49 V27
GMAT 2: 730 Q49 V40
GPA: 4
WE:Engineering (Consumer Electronics)
Send PM
Re: Despite improvements in treatment for asthma, the death rate from this [#permalink]
Increased urban pollution, improved recording of asthma deaths - serve as counter premise in the argument. Option C hits directly on the author's conclusion so isn't C the correct answer?
CR Moderator
Joined: 14 Dec 2013
Posts: 2413
Own Kudos [?]: 15266 [2]
Given Kudos: 26
Location: Germany
Schools:
GMAT 1: 780 Q50 V47
WE:Corporate Finance (Pharmaceuticals and Biotech)
Send PM
Re: Despite improvements in treatment for asthma, the death rate from this [#permalink]
2
Kudos
Expert Reply
nazgulISB wrote:
Increased urban pollution, improved recording of asthma deaths - serve as counter premise in the argument. Option C hits directly on the author's conclusion so isn't C the correct answer?


This is an assumption type question, not an inference type question. Option C could be an inference, i.e. option C follows FROM the conclusion, not that it leads TO the conclusion.
Director
Director
Joined: 04 Sep 2015
Posts: 552
Own Kudos [?]: 436 [1]
Given Kudos: 123
Location: India
WE:Information Technology (Computer Software)
Send PM
Re: Despite improvements in treatment for asthma, the death rate from this [#permalink]
1
Kudos
Despite improvements in treatment for asthma, the
death rate from this disease has doubled during the past
decade from its previous rate. Two possible explanations
for this increase have been offered. First, the recording of
deaths due to asthma has become more widespread and
accurate in the past decade than it had been previously.
Second, there has been an increase in urban pollution.
However, since the rate of deaths due to asthma has
increased dramatically even in cities with long-standing,
comprehensive medical records and with little or no urban
pollution, one must instead conclude that the cause of
increased deaths is the use of bronchial inhalers by asthma
sufferers to relieve their symptoms.

Which one of the following is an assumption on
which the argument depends?
(A) Urban pollution has not doubled in the past
decade.
The premise clearly mentions that the pollution has changed for some cities but not for all the cities,But if we do negation test that the pollution has doubled even then the argument provides premise that the deaths due to asthma has increased in the cities where the the pollution has not increased.
(B) Doctors and patients generally ignore the role
of allergies in asthma.
This option is also discussed in the argument premise that the deaths happen even in the cities where the mediacal documentation is appropriate and the patients are aware of the problems.
(C) Bronchial inhalers are unsafe, even when used
according to the recommended instructions.
this does not make a assumption question since this is a clear conclusion and not leading to the conclusion.
(D) The use of bronchial inhalers aggravates other
diseases that frequently occur among asthma
sufferers and that often lead to fatal outcomes
even when the asthma itself does not.
The information provided in the option is clearly linking the premises and the conclusion of the argument and addresses why the bronchial inhalers are unsafe.but if we do the negation test that bronchial inhalers does not aggravate other diseases.....,but this does not beark the argument as one can say that the inhalers might aggravate the issues related to asthma itself.
(E) Increased urban pollution, improved recording
of asthma deaths, and the use of bronchial
inhalers are the only possible explanations of
the increased death rate due to asthma.
Increased urban pollution ,improved records and use of bronchial inhalers are only possible explanations and if we negate this that the argument will be invalid since it then suggest that ther may be naother reason for the deaths..so this is the correct answer.
Moderator
Joined: 28 Mar 2017
Posts: 1090
Own Kudos [?]: 1970 [2]
Given Kudos: 200
Location: India
Concentration: Finance, Technology
GMAT 1: 730 Q49 V41
GPA: 4
Send PM
Re: Despite improvements in treatment for asthma, the death rate from this [#permalink]
2
Kudos
Despite improvements in treatment for asthma, the death rate from this disease has doubled during the past decade from its previous rate. Two possible explanations for this increase have been offered. First, the recording of deaths due to asthma has become more widespread and accurate in the past decade than it had been previously. Second, there has been an increase in urban pollution. However, since the rate of deaths due to asthma has increased dramatically even in cities with long-standing, comprehensive medical records and with little or no urban pollution, one must instead conclude that the cause of increased deaths is the use of bronchial inhalers by asthma sufferers to relieve their symptoms.

Which one of the following is an assumption on which the argument depends?

(A) Urban pollution has not doubled in the past decade. -Even if the population increased, the increase doesn't explain the cause of asthma related deaths
(B) Doctors and patients generally ignore the role of allergies in asthma. -out of scope
(C) Bronchial inhalers are unsafe, even when used according to the recommended instructions. -Then these wouldn't have been prescribed by the doctors
(D) The use of bronchial inhalers aggravates other diseases that frequently occur among asthma sufferers and that often lead to fatal outcomes even when the asthma itself does not. -other diseases are out of scope
(E) Increased urban pollution, improved recording of asthma deaths, and the use of bronchial inhalers are the only possible explanations of the increased death rate due to asthma. -Correct
Manager
Manager
Joined: 24 Mar 2018
Posts: 198
Own Kudos [?]: 41 [0]
Given Kudos: 288
Location: India
GMAT 1: 680 Q50 V31
Send PM
Re: Despite improvements in treatment for asthma, the death rate from this [#permalink]
VeritasKarishma nightblade354 gmat1393 AjiteshArun GMATNinja
I choose D
why D is wrong ?
Second reason has been contradicted already so how it can be the reason
Retired Moderator
Joined: 25 Apr 2018
Posts: 654
Own Kudos [?]: 2220 [0]
Given Kudos: 199
GMAT 1: 680 Q49 V34
Send PM
Re: Despite improvements in treatment for asthma, the death rate from this [#permalink]
teaserbae wrote:
VeritasKarishma nightblade354 gmat1393 AjiteshArun GMATNinja
I choose D
why D is wrong ?
Second reason has been contradicted already so how it can be the reason

The use of bronchial inhalers aggravating other diseases is out of scope of our Argument.Remember the Conclusion is
Quote:
the cause of increased deaths is the use of bronchial inhalers by asthma sufferers to relieve their symptoms.

Thats why option D is out of scope of our argument. In addition, you can see the detailed explanation at this link https://gmatclub.com/forum/despite-impr ... l#p1244388

VeritasKarishma DavidTutorexamPAL Please shed your views on option D.
Manager
Manager
Joined: 24 Mar 2018
Posts: 198
Own Kudos [?]: 41 [0]
Given Kudos: 288
Location: India
GMAT 1: 680 Q50 V31
Send PM
Re: Despite improvements in treatment for asthma, the death rate from this [#permalink]
gmat1393 wrote:
teaserbae wrote:
VeritasKarishma nightblade354 gmat1393 AjiteshArun GMATNinja
I choose D
why D is wrong ?
Second reason has been contradicted already so how it can be the reason

The use of bronchial inhalers aggravating other diseases is out of scope of our Argument.Remember the Conclusion is
Quote:
the cause of increased deaths is the use of bronchial inhalers by asthma sufferers to relieve their symptoms.

Thats why option D is out of scope of our argument. In addition, you can see the detailed explanation at this link https://gmatclub.com/forum/despite-impr ... l#p1244388

VeritasKarishma DavidTutorexamPAL Please shed your views on option D.


Can you please bried how D is out of scope ?
examPAL Representative
Joined: 07 Dec 2017
Posts: 1050
Own Kudos [?]: 1777 [0]
Given Kudos: 26
Send PM
Re: Despite improvements in treatment for asthma, the death rate from this [#permalink]
teaserbae wrote:
gmat1393 wrote:
teaserbae wrote:
VeritasKarishma nightblade354 gmat1393 AjiteshArun GMATNinja
I choose D
why D is wrong ?
Second reason has been contradicted already so how it can be the reason

The use of bronchial inhalers aggravating other diseases is out of scope of our Argument.Remember the Conclusion is
Quote:
the cause of increased deaths is the use of bronchial inhalers by asthma sufferers to relieve their symptoms.

Thats why option D is out of scope of our argument. In addition, you can see the detailed explanation at this link https://gmatclub.com/forum/despite-impr ... l#p1244388

VeritasKarishma DavidTutorexamPAL Please shed your views on option D.


Can you please bried how D is out of scope ?


An assumption on which an argument depends is something that must b true in order for the argument to be true: in other words, it is impossible that the argument be true and the assumption be wrong.
D certainly strengthens the argument, but is it an assumption? well, is it possible that the argument be true (the cause of asthma deaths is bronchial inhalers) but (D) be wrong (bronchial inhalers do NOT increase other diseases)? Absolutely! It may be that the inhalers have some totally different problem (maybe they're poisonous?? maybe they just don't work) unrelated to aggravating other diseases, and that the conclusion (inhalers > asthma) is true. Hence, D is not an assumption upon which the argument depends.
CEO
CEO
Joined: 07 Mar 2019
Posts: 2552
Own Kudos [?]: 1813 [0]
Given Kudos: 763
Location: India
WE:Sales (Energy and Utilities)
Send PM
Re: Despite improvements in treatment for asthma, the death rate from this [#permalink]
Despite improvements in treatment for asthma, the death rate from this disease has doubled during the past decade from its previous rate. Two possible explanations for this increase have been offered. First, the recording of deaths due to asthma has become more widespread and accurate in the past decade than it had been previously. Second, there has been an increase in urban pollution. However, since the rate of deaths due to asthma has increased dramatically even in cities with long-standing, comprehensive medical records and with little or no urban pollution, one must instead conclude that the cause of increased deaths is the use of bronchial inhalers by asthma sufferers to relieve their symptoms.

Which one of the following is an assumption on which the argument depends?

(A) Urban pollution has not doubled in the past decade. - WRONG.

(B) Doctors and patients generally ignore the role of allergies in asthma. - WRONG.

(C) Bronchial inhalers are unsafe, even when used according to the recommended instructions. - WRONG.

(D) The use of bronchial inhalers aggravates other diseases that frequently occur among asthma sufferers and that often lead to fatal outcomes even when the asthma itself does not. - WRONG. Had the question stem asked for strengthener this would have been the answer in the sense that bronchial inhalers are the cause of increase. But it is not an absolute necessity.

(E) Increased urban pollution, improved recording of asthma deaths, and the use of bronchial inhalers are the only possible explanations of the increased death rate due to asthma. - CORRECT. If the passage remains so as it is then bronchial inhalers are the cause because the other two are not.

D is trap that most would fell for as i did.

Answer E.
Intern
Intern
Joined: 29 Aug 2022
Posts: 1
Own Kudos [?]: 0 [0]
Given Kudos: 1
Send PM
Re: Despite improvements in treatment for asthma, the death rate from this [#permalink]
Gmat Ninja Can you please explain this ?
I had chosen option D
GMAT Club Verbal Expert
Joined: 13 Aug 2009
Status: GMAT/GRE/LSAT tutors
Posts: 6917
Own Kudos [?]: 63652 [0]
Given Kudos: 1773
Location: United States (CO)
GMAT 1: 780 Q51 V46
GMAT 2: 800 Q51 V51
GRE 1: Q170 V170

GRE 2: Q170 V170
Send PM
Re: Despite improvements in treatment for asthma, the death rate from this [#permalink]
Expert Reply
kuljitr wrote:
Gmat Ninja Can you please explain this ?
I had chosen option D

An assumption on which the argument depends is something that MUST be true in order for the author to reach his/her conclusion.

So, do we absolutely NEED the information in (D) to conclude that "the cause of increased deaths is the use of bronchial inhalers by asthma sufferers to relieve their symptoms"?

Here's (D):
Quote:
(D) The use of bronchial inhalers aggravates other diseases that frequently occur among asthma sufferers and that often lead to fatal outcomes even when the asthma itself does not.

(D) gives us a reason why inhalers might be causing deaths. But we don't need this to be the exact issue with inhalers -- there could be a million other reasons why inhalers are to blame for the deaths (e.g., perhaps someone is poisoning the inhalers, or perhaps people use inhalers improperly and die as a result). The argument doesn't absolutely require that the inhalers aggravate other diseases, so (D) is not an assumption on which the argument depends.

Eliminate (D).

Here's (E):
Quote:
(E) Increased urban pollution, improved recording of asthma deaths, and the use of bronchial inhalers are the only possible explanations of the increased death rate due to asthma.

In the passage, the author rules out urban pollution and improved recording of asthma deaths as causes of the increased death rate.

From this, he/she says that "one must instead conclude that the cause of increased deaths is the use of bronchial inhalers by asthma sufferers to relieve their symptoms."

But what if there are other things that might be causing the increased death rate? Then one wouldn't HAVE to conclude that inhalers are the problem.

So, to say that one MUST conclude that the inhalers are to blame, we MUST assume that there aren't any other potential explanations out there.

That's why (E) is an assumption on which the argument depends.

I hope that helps!
Intern
Intern
Joined: 12 Mar 2021
Posts: 11
Own Kudos [?]: 28 [0]
Given Kudos: 44
Location: India
GPA: 3
Send PM
Re: Despite improvements in treatment for asthma, the death rate from this [#permalink]
IMO E.

It must be true for the conclusion to be properly drawn

nakib77 wrote:
Despite improvements in treatment for asthma, the death rate from this disease has doubled during the past decade from its previous rate. Two possible explanations for this increase have been offered. First, the recording of deaths due to asthma has become more widespread and accurate in the past decade than it had been previously. Second, there has been an increase in urban pollution. However, since the rate of deaths due to asthma has increased dramatically even in cities with long-standing, comprehensive medical records and with little or no urban pollution, one must instead conclude that the cause of increased deaths is the use of bronchial inhalers by asthma sufferers to relieve their symptoms.

Which one of the following is an assumption on which the argument depends?

(A) Urban pollution has not doubled in the past decade.

(B) Doctors and patients generally ignore the role of allergies in asthma.

(C) Bronchial inhalers are unsafe, even when used according to the recommended instructions.

(D) The use of bronchial inhalers aggravates other diseases that frequently occur among asthma sufferers and that often lead to fatal outcomes even when the asthma itself does not.

(E) Increased urban pollution, improved recording of asthma deaths, and the use of bronchial inhalers are the only possible explanations of the increased death rate due to asthma.


"Despite improvements in treatment for asthma" EXCEPT Question
Intern
Intern
Joined: 29 Sep 2023
Posts: 30
Own Kudos [?]: 2 [0]
Given Kudos: 23
Location: Korea, Republic of
Concentration: Leadership, Strategy
GMAT 1: 750 Q50 V41
GPA: 3.6
Send PM
Re: Despite improvements in treatment for asthma, the death rate from this [#permalink]
Quote:
Despite improvements in treatment for asthma, the death rate from this disease has doubled during the past decade from its previous rate. Two possible explanations for this increase have been offered. First, the recording of deaths due to asthma has become more widespread and accurate in the past decade than it had been previously. Second, there has been an increase in urban pollution. However, since the rate of deaths due to asthma has increased dramatically even in cities with long-standing, comprehensive medical records and with little or no urban pollution, one must instead conclude that the cause of increased deaths is the use of bronchial inhalers by asthma sufferers to relieve their symptoms.

Which one of the following is an assumption on which the argument depends?


Type: Argument
Conclusion: the cause of increased deaths is the use of bronchial inhalers by asthma sufferers to relieve their symptoms
Premise: the rate of deaths due to asthma has increased dramatically even in cities with long-standing, comprehensive medical records and with little or no urban pollution
Answer: (E) Increased urban pollution, improved recording of asthma deaths, and the use of bronchial inhalers are the only possible explanations of the increased death rate due to asthma.

Why: There is a disconnection between the premise and the conclusion. This happens when there is a BIG assumption.
GMAT Club Bot
Re: Despite improvements in treatment for asthma, the death rate from this [#permalink]
Moderators:
GMAT Club Verbal Expert
6917 posts
GMAT Club Verbal Expert
238 posts
CR Forum Moderator
832 posts

Powered by phpBB © phpBB Group | Emoji artwork provided by EmojiOne